Prove a function is a norm?












0












$begingroup$


I want to show that $${||x||}_{infty} = max{{|x_1|,|x_1|,...,|x_n|}}$$
is a norm.



Where the properties of a norm are





  1. ${||x||} ge 0$ for all $x$ in${mathbb R}^n$


  2. ${||x||} = 0$ when $x=0$


  3. ${||ax||} = |a|{||x||}$ for all $a$ in ${mathbb R}$

  4. $||x+y|| le ||x||+||y||$


I think I intuitively understand that 1-3 hold, but I'm not certain how to actually prove this. I'm also not sure how to think about 4. What is $y$ in this case?










share|cite|improve this question











$endgroup$












  • $begingroup$
    3. follows since $||ax||=max{|ax_1|, ... , |ax_n|}=a max{|x_1|, ... , |x_n|}$ and 4. follows from the triangle inequality on $mathbb{R}$.
    $endgroup$
    – Mustafa Said
    Jan 25 at 2:40


















0












$begingroup$


I want to show that $${||x||}_{infty} = max{{|x_1|,|x_1|,...,|x_n|}}$$
is a norm.



Where the properties of a norm are





  1. ${||x||} ge 0$ for all $x$ in${mathbb R}^n$


  2. ${||x||} = 0$ when $x=0$


  3. ${||ax||} = |a|{||x||}$ for all $a$ in ${mathbb R}$

  4. $||x+y|| le ||x||+||y||$


I think I intuitively understand that 1-3 hold, but I'm not certain how to actually prove this. I'm also not sure how to think about 4. What is $y$ in this case?










share|cite|improve this question











$endgroup$












  • $begingroup$
    3. follows since $||ax||=max{|ax_1|, ... , |ax_n|}=a max{|x_1|, ... , |x_n|}$ and 4. follows from the triangle inequality on $mathbb{R}$.
    $endgroup$
    – Mustafa Said
    Jan 25 at 2:40
















0












0








0





$begingroup$


I want to show that $${||x||}_{infty} = max{{|x_1|,|x_1|,...,|x_n|}}$$
is a norm.



Where the properties of a norm are





  1. ${||x||} ge 0$ for all $x$ in${mathbb R}^n$


  2. ${||x||} = 0$ when $x=0$


  3. ${||ax||} = |a|{||x||}$ for all $a$ in ${mathbb R}$

  4. $||x+y|| le ||x||+||y||$


I think I intuitively understand that 1-3 hold, but I'm not certain how to actually prove this. I'm also not sure how to think about 4. What is $y$ in this case?










share|cite|improve this question











$endgroup$




I want to show that $${||x||}_{infty} = max{{|x_1|,|x_1|,...,|x_n|}}$$
is a norm.



Where the properties of a norm are





  1. ${||x||} ge 0$ for all $x$ in${mathbb R}^n$


  2. ${||x||} = 0$ when $x=0$


  3. ${||ax||} = |a|{||x||}$ for all $a$ in ${mathbb R}$

  4. $||x+y|| le ||x||+||y||$


I think I intuitively understand that 1-3 hold, but I'm not certain how to actually prove this. I'm also not sure how to think about 4. What is $y$ in this case?







functions norm






share|cite|improve this question















share|cite|improve this question













share|cite|improve this question




share|cite|improve this question








edited Jan 25 at 5:28









Chinnapparaj R

5,7032928




5,7032928










asked Jan 25 at 2:33









Chemical EngineerChemical Engineer

597




597












  • $begingroup$
    3. follows since $||ax||=max{|ax_1|, ... , |ax_n|}=a max{|x_1|, ... , |x_n|}$ and 4. follows from the triangle inequality on $mathbb{R}$.
    $endgroup$
    – Mustafa Said
    Jan 25 at 2:40




















  • $begingroup$
    3. follows since $||ax||=max{|ax_1|, ... , |ax_n|}=a max{|x_1|, ... , |x_n|}$ and 4. follows from the triangle inequality on $mathbb{R}$.
    $endgroup$
    – Mustafa Said
    Jan 25 at 2:40


















$begingroup$
3. follows since $||ax||=max{|ax_1|, ... , |ax_n|}=a max{|x_1|, ... , |x_n|}$ and 4. follows from the triangle inequality on $mathbb{R}$.
$endgroup$
– Mustafa Said
Jan 25 at 2:40






$begingroup$
3. follows since $||ax||=max{|ax_1|, ... , |ax_n|}=a max{|x_1|, ... , |x_n|}$ and 4. follows from the triangle inequality on $mathbb{R}$.
$endgroup$
– Mustafa Said
Jan 25 at 2:40












2 Answers
2






active

oldest

votes


















-1












$begingroup$

For 1. $max = |x_i| geq 0$ by the definition of absolute value, where $i$ is corresponds to the max component.
For 2. let $||x||=0 rightarrow 0 = max geq |x_i| geq 0 forall x_i rightarrow x=0$. Now let $x=0 rightarrow forall x_i, x_i = 0 rightarrow max = 0 rightarrow ||x||=0$.
For 3. $||ax|| = max |ax| = |a||x_i| = |a|max |x| = |a|||x||$, where $|x_i|$ is the max element.
For 4. $||x+y|| = |x_i+y_i| leq |x_i| + |y_i| leq max x + max y = ||x|| + ||y||$, where $|x_i + y_i|$ is the max element of $|x+y|$ (abs applied componentwise).






share|cite|improve this answer









$endgroup$





















    0












    $begingroup$

    To prove 4., notice that



    $$ ||x+y||_{infty} = |x_l+y_l | $$



    for some $l$ by the definition of max. Now apply the usual triangle inequality: $|x+y| leq |x| + |y| $ and since for example $|x_l| leq max |x_i| = ||x|| $, the result follows






    share|cite|improve this answer









    $endgroup$













      Your Answer





      StackExchange.ifUsing("editor", function () {
      return StackExchange.using("mathjaxEditing", function () {
      StackExchange.MarkdownEditor.creationCallbacks.add(function (editor, postfix) {
      StackExchange.mathjaxEditing.prepareWmdForMathJax(editor, postfix, [["$", "$"], ["\\(","\\)"]]);
      });
      });
      }, "mathjax-editing");

      StackExchange.ready(function() {
      var channelOptions = {
      tags: "".split(" "),
      id: "69"
      };
      initTagRenderer("".split(" "), "".split(" "), channelOptions);

      StackExchange.using("externalEditor", function() {
      // Have to fire editor after snippets, if snippets enabled
      if (StackExchange.settings.snippets.snippetsEnabled) {
      StackExchange.using("snippets", function() {
      createEditor();
      });
      }
      else {
      createEditor();
      }
      });

      function createEditor() {
      StackExchange.prepareEditor({
      heartbeatType: 'answer',
      autoActivateHeartbeat: false,
      convertImagesToLinks: true,
      noModals: true,
      showLowRepImageUploadWarning: true,
      reputationToPostImages: 10,
      bindNavPrevention: true,
      postfix: "",
      imageUploader: {
      brandingHtml: "Powered by u003ca class="icon-imgur-white" href="https://imgur.com/"u003eu003c/au003e",
      contentPolicyHtml: "User contributions licensed under u003ca href="https://creativecommons.org/licenses/by-sa/3.0/"u003ecc by-sa 3.0 with attribution requiredu003c/au003e u003ca href="https://stackoverflow.com/legal/content-policy"u003e(content policy)u003c/au003e",
      allowUrls: true
      },
      noCode: true, onDemand: true,
      discardSelector: ".discard-answer"
      ,immediatelyShowMarkdownHelp:true
      });


      }
      });














      draft saved

      draft discarded


















      StackExchange.ready(
      function () {
      StackExchange.openid.initPostLogin('.new-post-login', 'https%3a%2f%2fmath.stackexchange.com%2fquestions%2f3086632%2fprove-a-function-is-a-norm%23new-answer', 'question_page');
      }
      );

      Post as a guest















      Required, but never shown

























      2 Answers
      2






      active

      oldest

      votes








      2 Answers
      2






      active

      oldest

      votes









      active

      oldest

      votes






      active

      oldest

      votes









      -1












      $begingroup$

      For 1. $max = |x_i| geq 0$ by the definition of absolute value, where $i$ is corresponds to the max component.
      For 2. let $||x||=0 rightarrow 0 = max geq |x_i| geq 0 forall x_i rightarrow x=0$. Now let $x=0 rightarrow forall x_i, x_i = 0 rightarrow max = 0 rightarrow ||x||=0$.
      For 3. $||ax|| = max |ax| = |a||x_i| = |a|max |x| = |a|||x||$, where $|x_i|$ is the max element.
      For 4. $||x+y|| = |x_i+y_i| leq |x_i| + |y_i| leq max x + max y = ||x|| + ||y||$, where $|x_i + y_i|$ is the max element of $|x+y|$ (abs applied componentwise).






      share|cite|improve this answer









      $endgroup$


















        -1












        $begingroup$

        For 1. $max = |x_i| geq 0$ by the definition of absolute value, where $i$ is corresponds to the max component.
        For 2. let $||x||=0 rightarrow 0 = max geq |x_i| geq 0 forall x_i rightarrow x=0$. Now let $x=0 rightarrow forall x_i, x_i = 0 rightarrow max = 0 rightarrow ||x||=0$.
        For 3. $||ax|| = max |ax| = |a||x_i| = |a|max |x| = |a|||x||$, where $|x_i|$ is the max element.
        For 4. $||x+y|| = |x_i+y_i| leq |x_i| + |y_i| leq max x + max y = ||x|| + ||y||$, where $|x_i + y_i|$ is the max element of $|x+y|$ (abs applied componentwise).






        share|cite|improve this answer









        $endgroup$
















          -1












          -1








          -1





          $begingroup$

          For 1. $max = |x_i| geq 0$ by the definition of absolute value, where $i$ is corresponds to the max component.
          For 2. let $||x||=0 rightarrow 0 = max geq |x_i| geq 0 forall x_i rightarrow x=0$. Now let $x=0 rightarrow forall x_i, x_i = 0 rightarrow max = 0 rightarrow ||x||=0$.
          For 3. $||ax|| = max |ax| = |a||x_i| = |a|max |x| = |a|||x||$, where $|x_i|$ is the max element.
          For 4. $||x+y|| = |x_i+y_i| leq |x_i| + |y_i| leq max x + max y = ||x|| + ||y||$, where $|x_i + y_i|$ is the max element of $|x+y|$ (abs applied componentwise).






          share|cite|improve this answer









          $endgroup$



          For 1. $max = |x_i| geq 0$ by the definition of absolute value, where $i$ is corresponds to the max component.
          For 2. let $||x||=0 rightarrow 0 = max geq |x_i| geq 0 forall x_i rightarrow x=0$. Now let $x=0 rightarrow forall x_i, x_i = 0 rightarrow max = 0 rightarrow ||x||=0$.
          For 3. $||ax|| = max |ax| = |a||x_i| = |a|max |x| = |a|||x||$, where $|x_i|$ is the max element.
          For 4. $||x+y|| = |x_i+y_i| leq |x_i| + |y_i| leq max x + max y = ||x|| + ||y||$, where $|x_i + y_i|$ is the max element of $|x+y|$ (abs applied componentwise).







          share|cite|improve this answer












          share|cite|improve this answer



          share|cite|improve this answer










          answered Jan 25 at 2:50









          lightxbulblightxbulb

          1,115311




          1,115311























              0












              $begingroup$

              To prove 4., notice that



              $$ ||x+y||_{infty} = |x_l+y_l | $$



              for some $l$ by the definition of max. Now apply the usual triangle inequality: $|x+y| leq |x| + |y| $ and since for example $|x_l| leq max |x_i| = ||x|| $, the result follows






              share|cite|improve this answer









              $endgroup$


















                0












                $begingroup$

                To prove 4., notice that



                $$ ||x+y||_{infty} = |x_l+y_l | $$



                for some $l$ by the definition of max. Now apply the usual triangle inequality: $|x+y| leq |x| + |y| $ and since for example $|x_l| leq max |x_i| = ||x|| $, the result follows






                share|cite|improve this answer









                $endgroup$
















                  0












                  0








                  0





                  $begingroup$

                  To prove 4., notice that



                  $$ ||x+y||_{infty} = |x_l+y_l | $$



                  for some $l$ by the definition of max. Now apply the usual triangle inequality: $|x+y| leq |x| + |y| $ and since for example $|x_l| leq max |x_i| = ||x|| $, the result follows






                  share|cite|improve this answer









                  $endgroup$



                  To prove 4., notice that



                  $$ ||x+y||_{infty} = |x_l+y_l | $$



                  for some $l$ by the definition of max. Now apply the usual triangle inequality: $|x+y| leq |x| + |y| $ and since for example $|x_l| leq max |x_i| = ||x|| $, the result follows







                  share|cite|improve this answer












                  share|cite|improve this answer



                  share|cite|improve this answer










                  answered Jan 25 at 2:44









                  Jimmy SabaterJimmy Sabater

                  3,013325




                  3,013325






























                      draft saved

                      draft discarded




















































                      Thanks for contributing an answer to Mathematics Stack Exchange!


                      • Please be sure to answer the question. Provide details and share your research!

                      But avoid



                      • Asking for help, clarification, or responding to other answers.

                      • Making statements based on opinion; back them up with references or personal experience.


                      Use MathJax to format equations. MathJax reference.


                      To learn more, see our tips on writing great answers.




                      draft saved


                      draft discarded














                      StackExchange.ready(
                      function () {
                      StackExchange.openid.initPostLogin('.new-post-login', 'https%3a%2f%2fmath.stackexchange.com%2fquestions%2f3086632%2fprove-a-function-is-a-norm%23new-answer', 'question_page');
                      }
                      );

                      Post as a guest















                      Required, but never shown





















































                      Required, but never shown














                      Required, but never shown












                      Required, but never shown







                      Required, but never shown

































                      Required, but never shown














                      Required, but never shown












                      Required, but never shown







                      Required, but never shown







                      Popular posts from this blog

                      Mario Kart Wii

                      What does “Dominus providebit” mean?

                      Antonio Litta Visconti Arese